Geometry Again!

Following are some geometry questions I couldnot solve. They may be from other olympiads, and if their solutions are available online please point me to them.

  1. For a pentagon \(ABCDE,\ AE=ED,\ AB+CD=BC,\) and \(\angle BAE + \angle CDE=180 ^{\circ} \). P.T. \(\angle AED=2\angle BEC\).

  2. Let P be a point inside a triangle with C=90\angle C=90 ^{\circ} such that AP=ACAP=AC, and let M be the midpt of AB and CH be the altitude. Prove that PM bisects BPH\angle BPH iff A=60\angle A=60 ^{\circ}.

#Geometry #EuclideanGeometry #OlympiadMath #Puregeometry

Note by Megh Parikh
7 years, 3 months ago

No vote yet
1 vote

  Easy Math Editor

This discussion board is a place to discuss our Daily Challenges and the math and science related to those challenges. Explanations are more than just a solution — they should explain the steps and thinking strategies that you used to obtain the solution. Comments should further the discussion of math and science.

When posting on Brilliant:

  • Use the emojis to react to an explanation, whether you're congratulating a job well done , or just really confused .
  • Ask specific questions about the challenge or the steps in somebody's explanation. Well-posed questions can add a lot to the discussion, but posting "I don't understand!" doesn't help anyone.
  • Try to contribute something new to the discussion, whether it is an extension, generalization or other idea related to the challenge.
  • Stay on topic — we're all here to learn more about math and science, not to hear about your favorite get-rich-quick scheme or current world events.

MarkdownAppears as
*italics* or _italics_ italics
**bold** or __bold__ bold

- bulleted
- list

  • bulleted
  • list

1. numbered
2. list

  1. numbered
  2. list
Note: you must add a full line of space before and after lists for them to show up correctly
paragraph 1

paragraph 2

paragraph 1

paragraph 2

[example link](https://brilliant.org)example link
> This is a quote
This is a quote
    # I indented these lines
    # 4 spaces, and now they show
    # up as a code block.

    print "hello world"
# I indented these lines
# 4 spaces, and now they show
# up as a code block.

print "hello world"
MathAppears as
Remember to wrap math in \( ... \) or \[ ... \] to ensure proper formatting.
2 \times 3 2×3 2 \times 3
2^{34} 234 2^{34}
a_{i-1} ai1 a_{i-1}
\frac{2}{3} 23 \frac{2}{3}
\sqrt{2} 2 \sqrt{2}
\sum_{i=1}^3 i=13 \sum_{i=1}^3
\sin \theta sinθ \sin \theta
\boxed{123} 123 \boxed{123}

Comments

For Problem 1. Consider the point FF in the ray BABA such that AF=CDAF=CD (AA between BB and FF), then triangles FAEFAE and CDECDE are congruent (using AE=EDAE=ED and BAE+CDE=180\angle BAE+\angle CDE=180^\circ. Note that the triangles CEBCEB and FEBFEB are congruent because EC=EFEC=EF and BC=AB+CD=AB+AF=BFBC=AB+CD=AB+AF=BF. Finally, BEC=BEF=BEA+AEF=BEA+DEC\angle BEC=\angle BEF=\angle BEA+\angle AEF=\angle BEA+\angle DEC therefore AED=2BEC\angle AED=2\angle BEC.

Jorge Tipe - 7 years, 3 months ago

Log in to reply

Thanks.

Megh Parikh - 7 years, 3 months ago

This problem wouldn't be so hard once you notice a property that PP carries from AC=APAC=AP.

WLOG say that HH is between A,MA,M. Since AC=AH×ABAC=AH\times AB, therefore AP=AHABAP=AH*AB which is enough to show that APHABPPHPB=APAB=ACAB\triangle APH\sim \triangle ABP\Rightarrow \frac {PH}{PB}=\frac {AP}{AB}=\frac {AC}{AB}. Since PMPM bisects BPH    PHPB=HMBM\angle BPH\iff \frac {PH}{PB}=\frac {HM}{BM} combining the results above we continue the equivalence:     ACAB=HMBM=2HMAB\iff \frac {AC}{AB}=\frac {HM}{BM}=\frac {2HM}{AB}     AC=2HM\iff AC=2HM.

Instead to computing for HMHM, we can geometrically show that AC=2HM    AAC=2HM\iff \angle A. To relate ACAC and HMHM, we introduce the midpoint of ACAC(I actually originally connected GHGH, but then I found a simpler way). Since AGM=CHM=90\angle AGM=\angle CHM=90(CM=AMCM=AM) and AG=AC2=HMAG=\frac {AC}{2}=HM, by HLHL congruence we have GAM=CMA=ACMCAB=60\angle GAM=\angle CMA=\angle ACM\Rightarrow \angle CAB=60. Q.E.D

For the first one I have the same proof as Jorge :)

Xuming Liang - 7 years, 3 months ago

Log in to reply

As a crazy thought for this problem, we could also argue by considering when Apollonius' circle of H,B;MH,B;M and the circle centered at AA with radius ACAC intersect.

Xuming Liang - 7 years, 3 months ago

Thanks.

Megh Parikh - 7 years, 3 months ago
×

Problem Loading...

Note Loading...

Set Loading...